0 Daumen
214 Aufrufe

Ich möchte beweisen, dass \( (1+\frac{1}{n})^n ≤ 4 \) für \(n∈ℕ, n≥1\) gilt. Dabei sollen die folgenden Aussagen benutzt werden:

(1)  \( 2^{n-1} ≤ n! \) ,

(2)  \( \sum\limits_{k=0}^{n}{x^k} = \frac{1-x^{n+1}}{1-x}\) (\( x ≠ 1\)),

und Binomischer Lehrsatz.


Mein Ansatz ist vollständige Induktion. Also Induktionsanfang \(n=1 \):

\((1+ \frac{1}{1})^1 = 2 ≤ 4\)

Dann der Induktionsschritt \( n → n+1\):

\( 4 ≥ (1+ \frac{1}{n+1})^{n+1} = \sum\limits_{k=0}^{n+1}{ \begin{pmatrix}n+1\\k\end{pmatrix} (\frac{1}{n+1})^{k}}\) ...

Und da hört es auch leider schon auf. Ich bin mir an der Stelle nicht sicher wie ich weiter verfahren soll. Ich habe unter anderem versucht mit (1) abzuschätzen oder umzuformen um (2) anwenden zu können - aber ohne Erfolg.

Weis jemand wie man die Aussage beweisen kann? Für jede Hilfe bin ich dankbar!

Avatar von

Man kann die Behauptung auch direkt, also ohne Induktion, zeigen. Verwende dazu die Ungleichung
\(\displaystyle\binom nk\le\frac{n^k}{k!}\) für \(0\le k\le n\) und anschließend deine Aussagen (1) und (2).

1 Antwort

+1 Daumen

Aloha :)

Mit den genannten Methoden kann man sogar zeigen, dass der Term kleiner als \(3\) ist.

$$a_n=\left(1+\frac{1}{n}\right)^n=\sum\limits_{k=0}^n\binom{n}{k}1^{n-k}\cdot\left(\frac{1}{n}\right)^k=\sum\limits_{k=0}^n\binom{n}{k}\frac{1}{n^k}=\sum\limits_{k=0}^n\frac{n!}{k!(n-k)!}\frac{1}{n^k}$$$$\phantom{a_n}=\sum\limits_{k=0}^n\frac{n\cdot(n-1)\cdots(n-k+1)}{k!\cdot n^k}=\sum\limits_{k=0}^n\frac{1}{k!}\,\frac{n}{n}\,\frac{n-1}{n}\,\frac{n-2}{n}\cdots\frac{n-k+1}{n}$$$$\phantom{a_n}\le\sum\limits_{k=0}^n\frac{1}{k!}=1+\frac{1}{1!}+\frac{1}{2!}+\frac{1}{3!}+\frac{1}{4!}+\cdots+\frac{1}{(n-1)!}+\frac{1}{n!}$$$$\phantom{a_n}\le1+\frac{1}{1}+\frac{1}{1\cdot2}+\frac{1}{2\cdot3}+\frac{1}{3\cdot4}+\cdots+\frac{1}{(n-2)(n-1)}+\frac{1}{(n-1)n}$$$$\phantom{a_n}=2\!+\!\left(\frac{1}{1}\!-\!\frac{1}{2}\right)\!+\!\left(\frac{1}{2}\!-\!\frac{1}{3}\right)\!+\!\left(\frac{1}{3}\!-\!\frac{1}{4}\right)\!+\!\cdots\!+\!\left(\frac{1}{n-2}\!-\!\frac{1}{n-1}\right)\!+\!\left(\frac{1}{n-1}\!-\!\frac{1}{n}\right)$$$$\phantom{a_n}=2+\frac{1}{1}-\frac{1}{n}=3-\frac{1}{n}<3$$

Avatar von 148 k 🚀

Ein anderes Problem?

Stell deine Frage

Willkommen bei der Mathelounge! Stell deine Frage einfach und kostenlos

x
Made by a lovely community